Mistake in a question? 19 Base pairs

is something wrong with this question, or am i not seeing something? :sweat_smile::relieved:

Answer credited to @Asafmen

This question seems to have some missing information.

There is a possible mathematical solution for 3(B) and 5(C), however, it is not much help. Using trial and error is also not adequate.

This question is flawed, and is not worth looking into.